Difference between revisions of "2003 AMC 10A Problems/Problem 11"

 
(Redirected page to 2003 AMC 12A Problems/Problem 5)
 
(2 intermediate revisions by 2 users not shown)
Line 1: Line 1:
== Problem ==
+
#REDIRECT[[2003 AMC 12A Problems/Problem 5]]
The sum of the two 5-digit numbers <math>AMC10</math> and <math>AMC12</math> is <math>123422</math>. What is <math>A+M+C</math>?
 
 
 
<math> \mathrm{(A) \ } 10\qquad \mathrm{(B) \ } 11\qquad \mathrm{(C) \ } 12\qquad \mathrm{(D) \ } 13\qquad \mathrm{(E) \ } 14 </math>
 
 
 
== Solution ==
 
<math>M+M</math> is odd, so <math>C+C</math> must carry over, and therefore, <math>C=7</math>.
 
 
 
<math>A+A=12</math>, so <math>A=6</math>.
 
 
 
<math>M+M+1=3</math>, so <math>M=1</math>.
 
 
 
<math>7+6+1=14\Longrightarrow\mathrm{(E)}</math>
 
 
 
 
 
== See Also ==
 
* [[2003 AMC 10A Problems]]
 
* [[2003 AMC 10A Problems/Problem 12 | Next Problem]]
 
* [[2003 AMC 10A Problems/Problem 10 | Previous Problem]]
 

Latest revision as of 14:35, 30 July 2011